Vous êtes sur la page 1sur 84

www.insightsonindia.

com
www.insightsias.com
TEST 28 Solutions

Prelims 2016 Test 28


SOLUTIONS

1. The Union Cabinet has approved some key decisions during 2013-15
related to Bioethanol and Biodiesel for implementation of National
Policy on Biofuels in. Which of the following is among one of them?
a) Producing Ethanol from Molasses will be discouraged.
b) Ethanol should only be produced using non-food feestocks.
c) To promote Biodiesel, their prices will be administered by the
government.
d) None of the above

Solution: d)
Justification: The following decisions were approved.

Sugarcane or sugarcane juice may not be used for production of ethanol


and it be produced only from Molasses. So, (a) is incorrect.

Ethanol produced from other non-food feedstocks besides molasses like


cellulosic and ligno cellulosic materials and including petrochemical
route, may be allowed to be procured subject to meeting the relevant
BIS standards. So, (b) is incorrect.

Private biodiesel manufacturers should be acknowledged. , their


authorized dealers

Marketing rights be given to the Private biodiesel Manufacturers, their


authorised dealers and for direct sales to consumers.

The price of bio-diesel will be market determined. So, (c) is incorrect.

In view of above and also in the interest of meeting out the blending targets of
the Biofuel Policy, it is requested to kindly take necessary measures for
implementation of the above decisions for biodiesel blending in diesel as well
as use of B100 as standalone fuel subject to BIS standards and guidelines
prescribed by MoPNG.

Q Source: http://pib.nic.in/newsite/mbErel.aspx?relid=124076
www.insightsonindia.com
www.insightsias.com
TEST 28 Solutions

2. Consider the following about bills referred to a Joint Committee of the


Parliament.
1. A Joint Committee cannot recommend amendment of the bill, and
can only record its observations on the bill.
2. A Money bill cannot be referred to a Joint Committee of the Houses.

Which of the above is/are correct?

a) 1 only
b) 2 only
c) Both 1 and 2
d) None

Solution: b)
Justification: Statement 1: If a Bill is referred to a Select or a Joint
Committee, it considers the Bill clause-by-clause just as the House does.
Amendments can be moved to the various clauses by the members of the
Committee.

After the report of the Select or Joint Committee has been presented to the
House, the member-in-charge of the Bill usually moves the motion for
consideration of the Bill, as reported by the Select or Joint Committee, as the
case may be.

Statement 2: A Money Bill or a Financial Bill containing any of the provisions


calculated to make a Bill a Money Bill, however, cannot be referred to a Joint
Committee of the Houses. This is because RS has no powers of amending a
money bill.

Q Source: http://164.100.47.192/Loksabha/Legislation/Legislation.aspx
www.insightsonindia.com
www.insightsias.com
TEST 28 Solutions

3. The Coal resources of India are available in


1. Old Gondwana Formations of peninsular India
2. Young Tertiary formations of north-eastern region

Which of the above is/are correct?

a) 1 only
b) 2 only
c) Both 1 and 2
d) None

Solution: c)
Justification: Different regions in India contain rocks of all types belonging
to different geologic periods

India can be divided into several divisions such as Archean System, Dharwar
System, Cudappah system, Vindhyan system, Paleozoic,Mesozoic, Gondwana,
Deccan Trap, Tertiary and Alluvial.

Statement 1: India's geographical land area can be classified into Deccan trap,
Gondwana and Vindhyan.

This system contains famous Damuda and Panchet series which are famous
for coal deposits.

The important coal bearing areas of this series are Raniganj, Jharia, Karanpur,
and Bokaro of the Damodar basin in Odisha, and the Pench valley in
Chhattisgarh and Madhya Pradesh, the jhingurda coal seam (Chhattisgarh).

Statement 2: The Tertiary rock system belongs to Cenozoic era.

The final break-up of the Gondwana land occurred in this era and the Tethys
sea got lifted in the Himalayas.
http://www.gktoday.in/blog/indias-rock-formation-archean-dharwar-cudappah-vindhyan-
gondwana-and-tertiary-rocks/ - The_Tertiary_System The rocks of this era have valuable
deposits of petroleum and coal.

Also, Hard coal deposit spread over 27 major coalfields, are mainly confined to
eastern and south central parts of the the country. The lignite reserves stand at
a level around 36 billion tonnes, of which 90 % occur in the southern State of
Tamil Nadu.

Q Source: http://coal.gov.in/content/coal-indian-energy-choice
www.insightsonindia.com
www.insightsias.com
TEST 28 Solutions

4. Which of the following recommendations of Acworth Committee (1920-


21) recently became a matter of national debate?
a) Separation of railways budget from general budget
b) Having a Common Chief of Staff in the armed forces
c) Reducing the dominance of State-owned firms in the economy
d) Allowing only domiciles of a state to fight elections from that particular
state

Solution: a)
Learning: Pursuant to the recommendations of the Acworth Committee
(1920-21), Railway Finances were separated in 1924 for General Finances
primarily to secure stability for civil estimates by providing for an assured
contribution from Railway Revenues and also to introduce flexibility in the
administration of Railway Finances.

Recently NITI Ayog submitted a note to the GoI suggesting that this
separation be done away with to reduce populism in railways budgets.

You can read more at the Q Source.

Q Source: http://timesofindia.indiatimes.com/india/Scrap-railway-budget-
Niti-Aayog-panel/articleshow/52860225.cms

5. The Lucknow Pact (1916) resulted in


1. Formation of a notional body that would serve as the future
constituent assembly of India
2. Demand of Swaraj to the British government
3. Merger of moderates and extremists factions of Indian National
Congress (INC)
4. An agreement for coordinated action against British Rule by INC and
Muslim league

Select the correct answer using the codes below.


www.insightsonindia.com
www.insightsias.com
TEST 28 Solutions

a) 1 and 3 only
b) 3 and 4 only
c) 1, 2 and 4 only
d) 1, 2, 3 and 4

Solution: b)
Justification: Statement 1 and 2: These happened much after starting from
the Lahore session (Swaraj) and reaching to the formation of constituent
assembly after cabinet Mission plan (1946).

Statement 3 and 4: During the 1916 Congress session at Lucknow two major
events occurred. The divided Congress became united. An understanding for
joint action against the British was reached between the Congress and the
Muslim League and it was called the Lucknow Pact.

The signing of the Lucknow Pact by the Congress and the Muslim League in
1916 marked an important step in the Hindu-Muslim unity.

Q Source: 12th TN History Textbook

6. The Reserve Bank of India (RBI) regulates which of the following


aspects of Commercial Banks in India?
1. Issuance of licences for opening of banks
2. Expansion and approval of Indian banks to operate overseas
3. Monitoring maintenance of SLR and CRR by banks
4. Dealing with policy issues relating to customer service

Select the correct answer using the codes below.

a) 2 and 3 only
b) 1 and 4 only
c) 1, 2 and 3 only
d) 1, 2, 3 and 4

Solution: d)
www.insightsonindia.com
www.insightsias.com
TEST 28 Solutions

Justification: Statement 1 and 2: Licences for opening of banks;


Authorisations for opening of branches by banks in India, governing foreign
banks entry and expansion and approval of Indian banks to operate overseas
etc. are regulated by RBI.

Statement 3 and 4: Policy formulation, review and implementation on


Prudential Norms, Basel II and III frameworks, validation of quantitative
models on Credit, Market and Operational Risks, Stress testing, CRR, SLR
adherence etc are also seen by the RBI.

It also approves appointments of chief executive officers (private sector and


foreign banks) and their compensation packages.

Q Source: https://www.rbi.org.in/scripts/FS_Overview.aspx?fn=2

7. Indias Reusable Launch Vehicle-Technology Demonstrator (RLV-TD) is


beneficial as it would help to
a) Reduce future launch costs
b) Make satellites solely with the help of Green technology
c) Place satellites higher in the ionosphere
d) All of the above

Solution: a)
Learning: The cost of access to space is the major deterrent in space
exploration and space utilization. A reusable launch vehicle is the unanimous
solution to achieve low cost, reliable and on-demand space access.

Reusable Launch Vehicle-Technology Demonstration Program or RLV-TD is a


series of technology demonstration missions that have been considered as a
first step towards realizing a Two Stage To Orbit (TSTO) fully re-usable
vehicle.

A Winged Reusable Launch Vehicle technology Demonstrator (RLV-TD) has


been configured to act as a flying test bed to evaluate various technologies,
namely, hypersonic flight, autonomous landing, powered cruise flight and
hypersonic flight using air-breathing propulsion.
www.insightsonindia.com
www.insightsias.com
TEST 28 Solutions

These technologies will be developed in phases through a series of


experimental flights.

Q Source: ISRO Website

8. Uruguay round is mentioned in the context of


a) World Trade Organization (WTO)
b) International Monetary Fund (IMF)
c) World Economic Forum
d) World Health Organization (WHO)

Solution: a)
Learning: The Uruguay Round was the 8th round of multilateral trade
negotiations (MTN) conducted within the framework of the General
Agreement on Tariffs and Trade (GATT), spanning from 1986 to 1994 and
embracing 123 countries as "contracting parties".

The Round led to the creation of the World Trade Organization, with GATT
remaining as an integral part of the WTO agreements.

The broad mandate of the Round had been to extend GATT trade rules to
areas previously exempted as too difficult to liberalize (agriculture, textiles)
and increasingly important new areas previously not included (trade in
services, intellectual property, investment policy trade distortions)

Q Source: WTO Website

9. At the commencement of the first session after each general election to


the House of the People and at the commencement of the first session of
each year the President shall address both Houses of Parliament
assembled together and inform Parliament of the causes of its
summons. This is a
a) Constitutional provision
www.insightsonindia.com
www.insightsias.com
TEST 28 Solutions

b) Statutory provisions
c) Rules of Procedure of the House
d) Parliamentary Convention

Solution: a)
Learning: Article 87(1) of the Constitution provides for this.

This is the case of the first session after each general election to the Lok Sabha,
the President addresses both Houses of Parliament assembled together after
the Members have made and subscribed the oath or affirmation and the
Speaker has been elected.

It takes generally two days to complete these preliminaries. No other business


is transacted tilt the President has addressed both Houses of Parliament
assembled together and informed Parliament of the Governments agenda.

Q Source:
http://164.100.47.192/loksabha/writereaddata/our%20parliament/Opening%
20of%20parliament%20by%20the%20president.pdf

10. The largest potential for small hydro projects lies in


a) Himalayan states
b) Central India
c) Western India
d) Coastal belts

Solution: a)
Justification: Small hydro is often developed using existing dams or through
development of new dams whose primary purpose is river and lake water-level
control, or irrigation.
www.insightsonindia.com
www.insightsias.com
TEST 28 Solutions

So, small hydro projects require flowing streams, better if streams are
perennial and flow at a fast speed. Himalayan region provides a perfect
potential ground. So, (a) is correct.

In Coastal region, stream velocity becomes very low, so (d) cant be the
answer.

There are not many streams in Central India and Western India when we
compare them to Himalayan states. So, (c) and (b) arent the answer.

Also, states were irrigation canals are dominant small hydro can be generated.

Q Source: www.mnre.gov.in/schemes/grid-connected/small-hydro/

11. Arsenic is released in the environment as a part of its natural cycle as a


result of
1. Volcanic eruptions
2. Mining
3. Acidification of water bodies

Select the correct answer using the codes below.

a) 1 and 2 only
b) 1 and 3 only
c) 2 only
d) 1, 2 and 3

Solution: a)
Justification: The Arsenic cycle has been explained well in the image below.
www.insightsonindia.com
www.insightsias.com
TEST 28 Solutions

Q Source: http://jnuenvis.nic.in/arsenic.html

12. With reference to Indian freedom struggle, Anusilan Samiti, Jugantar


and Abhinav Bharat were
a) Revolutionary secret organizations
b) Organizations that managed the writing and publishing work of Indian
National Congress (INC) in Vernacular languages
c) Offshoots of Congress that relied on constitutional methods of struggle
against the British
d) Cultural organizations aimed at reviving Hinduism

Solution: a)
Learning: In the first half of the 20th century, revolutionary groups sprang
up mainly in Bengal, Maharashtra, Punjab and Madras.
www.insightsonindia.com
www.insightsias.com
TEST 28 Solutions

The revolutionaries were not satisfied with the methods of both the
moderates and extremists.

Hence, they started many revolutionary secret organizations. In Bengal


Anusilan Samiti and Jugantar were established.

In Maharashtra Savarkar brothers had set up Abhinava Bharat.

In the Madras Presidency, Bharathmatha Association was started by


Nilakanta Bramachari.

In London, at India House, Shyamji Krishna Verma gathered young


Indian nationalists like Madan Lal Dhingra, Savarkar, V.V.S. Iyer and
T.S.S.Rajan. Lala Hardyal set up the Ghadar Party in USA to organise
revolutionary activities from outside India.

Q Source: 12th TN History Textbook

13. Many development and industrial projects such as erection of dams,


mining, and construction of industries or roads require diversion of
forest land. Consider the regulatory aspects of it.
1. Any project proponent, government or private must apply for forest
clearance from Ministry of Environment, Forests and Climate
Change (MoEFCC), before the conversion of land takes place.
2. The compensation for the lost forest land is decided by the MoEFCC
and the regulators.

Which of the above is/are correct?

a) 1 only
b) 2 only
c) Both 1 and 2
d) None

Solution: c)
www.insightsonindia.com
www.insightsias.com
TEST 28 Solutions

Learning: With a cover of 23% of Geographical area of the country, forest in


India comprise of a number of diverse forest types and reserved areas
designated as National Parks and Wildlife Sanctuaries.

Statement 1: Compensatory Afforestation Fund Management and Planning


Authority (CAMPA) are meant to promote afforestation and regeneration
activities as a way of compensating for forest land diverted to non-forest uses.

Statement 2: This proposal is to be submitted through the concerned forest


department of the state government. If clearance is given, then compensation
for the lost forest land is also to be decided by the ministry and the regulators.

Q Source: http://indianexpress.com/article/explained/campa-afforestation-
bill-rajya-sabha-green-india-mission-narendra-modi-2817475/

14. Ubiquitous computing is an emerging domain referring to


a) Decentralization of computing power to human brain
b) Trend towards embedding microprocessors in everyday objects so they
can communicate information
c) Low cost supercomputing powers
d) Bridging the digital divide between rural and urban regions

Solution: b)
Learning: It is the growing trend towards embedding microprocessors in
everyday objects so they can communicate information.

The words pervasive and ubiquitous mean "existing everywhere." Pervasive


computing devices are completely connected and constantly available.

Pervasive computing relies on the convergence of wireless technologies,


advanced electronics and the Internet.

The image below will clear its meaning.


www.insightsonindia.com
www.insightsias.com
TEST 28 Solutions

Q Source: http://deity.gov.in/content/ubiquitous-computing

15. Niyamgiri Hills, Odisha, was in news sometime back due to


a) Reduction of the hill top into a flat mud base in a short time
b) Discovery of a dormant volcano in the hills
c) New variety of iron ore was found here
d) Mining affecting cultural rights of tribals

Solution: d)
Learning: Niyamgiri hills - home to Dongria Kondhs, a primitive tribal
group, a few hundred Kutia Kondhs and other forest-dwellers is considered
sacred by the indigenous tribes and others as it is the abode of Niyamraja,
their presiding deity.

Vedanta was to be given license to mine bauxite in the hills.


www.insightsonindia.com
www.insightsias.com
TEST 28 Solutions

But, SC intervened citing the violation of tribals cultural rights and ordered
conducting referendum against the mining order of the government.

After the unanimous resolution opposing mining was adopted, Vedanta had to
buckle down.

This is the first time an environmental referendum is conducted on a directive


by the Supreme Court to find out whether mining in Niyamgiri will
tantamount to an infringement of the religious, community and individual
rights of local forest-dwellers.

Q Source: http://www.thehindu.com/news/national/12th-gram-sabha-too-
votes-against-vedanta-mining/article5039304.ece

16. Which of the following core industries has seen a consistent decline in
production year on year for some time?
a) Steel
b) Natural Gas
c) Cement
d) Electricity

Solution: b)
Learning: The Natural Gas production (weight: 1.71 %) decreased by 6.9 % in
May, 2016 over May, 2015. Its cumulative index during April to May, 2016-17
declined by 6.9 % over the corresponding period of previous year. Moreover, it
has seen a consistent fall in output since 2011-12.

Best performing sector is electricity. (Please see the image below)


www.insightsonindia.com
www.insightsias.com
TEST 28 Solutions

Q Source: http://commerce.nic.in/DOC/PressRelease.aspx?Id=6346

17. Consider the following about Swaraj Party.


1. It was formed after the split within Congress in the Gaya session due
to the calling off of the Non-cooperation movement.
2. It demanded the setting up of responsible government in India with
the necessary changes in the Government of India Act of 1919.
3. Motilal Nehru and C.R. Das were associated with the party.

Select the correct answer using the codes below.

a) 1 only
b) 2 and 3 only
c) 1 and 3 only
d) 1, 2 and 3

Solution: d)
Justification: Statement 1: The suspension of the Non-Cooperation
Movement led to a split within Congress in the Gaya session of the Congress in
www.insightsonindia.com
www.insightsias.com
TEST 28 Solutions

December 1922. Leaders like Motilal Nehru and Chittranjan Das formed a
separate group within the Congress known as the Swaraj Party in January
1923.

Statement 2 and 3: The Swarajists wanted to contest the council elections and
wreck the government from within.

Elections to Legislative Councils were held in November 1923. In this, the


Swaraj Party gained impressive successes. In the Central Legislative Council
Motilal Nehru became the leader of the party whereas in Bengal the party was
headed by C.R. Das.

The Swaraj Party did several significant things in the Legislative Council
including demanding a responsible government.

Q Source: 12th TN History Textbook

18. Biodiversity Conservation & Rural Livelihood Improvement Project


(BCRLIP) is being implemented by
a) Ministry of Rural Development (MoRD)
b) Ministry of Environment, Forests and Climate Change
c) Ministry of Planning
d) Global Environment Facility, India Cell

Solution: b)
Learning: It aims at conserving Biodiversity in selected landscapes,
including wildlife protected areas/critical conservation areas while improving
rural livelihoods through participatory approaches.

Development of Joint Forest Management (JFM) and eco-development in


some states are models of new approaches to provide benefits to both
conservation and local communities.

The project intends to build on these models and expand lessons to other
globally significant sites in the country to strengthen linkages between
conservation and improving livelihoods of local communities that live in the
neighbourhood of biodiversity rich areas-as well as to enhance the local and
national economy.
www.insightsonindia.com
www.insightsias.com
TEST 28 Solutions

Q Source: MoEF Website

19. ChaakiyarKoothu and VeethiNaatakam are


a) Folk plays
b) Forms of tribal martial art
c) Heritage styles of construction popular among Nilgiri tribes
d) Royal theatrics mentioned in Sangam literature

Solution: a)
Learning: Indian folk literature holds out a strong and loud message for
other parts of the world where these art forms have disappeared thick and fast
in consonance with rapid industrialization and globalization.

Folk literature and folk art forms are not merely carriers of culture or
philosophical poems, but rather the expressions of strong self-
reflections and deep insights accrued therein.

Simple life, self-reflection and treading the path of the righteous


contained in traditions.

Again, folk traditions are not merely platforms for holding high moral
ground having no relevance to the present day reality.

Several folk plays like ChaakiyarKoothu and VeethiNaatakam are used


even today as satire plays and commentaries on the current social and
political reality.

Same holds true for many folk songs from the vast pages of Indian
literature.

Q Source: Ministry of Culture website

20. Posco India expected to start steel plant construction promptly


after signing the 2005 MoU with Odisha government. However, the
www.insightsonindia.com
www.insightsias.com
TEST 28 Solutions

company wasnt able to start construction for next few years and faced
several problems due to ecological and legal controversies. These were
due to
a) Enactment of the Scheduled Tribes and Other Traditional Forest
Dwellers (Recognition of Forest Rights) Act, 2006
b) Indulging in Illegal mining and flouting environmental norms.
c) The MoU was signed without the consent of the Government of India.
d) PESA 1996 areas are not allowed to entertain foreign mining firms.

Solution: a)
Learning: POSCO project has been a quite controversial project.

A year after the 2005 MoU was signed, India's parliament passed
Forests Rights Act, which directly affected Posco-India.
The law granted certain rights to forest-dwelling communities in India,
including the use of forest land and other resources.
Supreme Court examined the facts related to the state government of
Odisha's initiatives to enter into MoU to encourage economic growth in
the state, and whether such development meets the intent of ecological
and environmental laws of India.
It ruled in favour of the state of Odisha saying such laws cant apply
retrospectively to MOUs.
In 2010, NC Saxena committee visited Odisha and alerted MoEF about
the non-recognition of forest rights by the Government of Odisha and
violation of the Forest Rights Act, in the forest areas proposed to be
diverted for the POSCO India project.
The committee urged the Ministry of Environment and Forests of
Government of India to withdraw the clearance given to the State
Government for diversion of the forest land, which the Ministry did.
Meena Committee was appointed in 2011, and it too accused POSCO of
violating the laws. However, in 2011, the Ministry of Environment and
Forests of India gave final clearance to the Posco-India project.

Q Source: http://www.moef.nic.in/sites/default/files/Possco_prjt.pdf

21. The structural part of the Constitution is largely derived from the
www.insightsonindia.com
www.insightsias.com
TEST 28 Solutions

a) Cabinet Mission Plan


b) Lahore Session of the Indian National Congress (INC)
c) Regulating Act of 1858
d) Government of India Act of 1935

Solution: d)
Justification: Provincial autonomy, administrative structure, the
establishment of a Federal Court, provision for the establishment of a
"Federation of India", a representative government and many more features
were borrowed directly from the GoI Act 1935.

Learning: The philosophical part of the Constitution (the Fundamental


Rights and the Directive Principles of State Policy) derives its inspiration from
the American and Irish Constitutions respectively.

The political part of the Constitution (the principle of Cabinet Government


and the relations between the executive and the legislature) has been largely
drawn from the British Constitution.

Q Source: Indian Polity: M Laxmikanth

22. Which of the following is the primary objective of monetary policy


in India?
a) Maintaining low levels of unemployment
b) Appreciate the exchange value of rupee in the Forex markets
c) Maintaining price stability
d) Tackling stressed assets of banks

Solution: c)
Justification: The Reserve Bank of India (RBI) is vested with the
responsibility of conducting monetary policy with the primary objective of
maintaining price stability while keeping in mind the objective of growth. This
responsibility is explicitly mandated under the Reserve Bank of India Act, as
amended in 2016.

It is now transferred to the Monetary Policy Committee, which will also face
the same primary objective, i.e. to keep inflation within an acceptable band.
www.insightsonindia.com
www.insightsias.com
TEST 28 Solutions

If inflation is in control, prices will be relatively stable.

Q Source: RBI Website

23.Communal representation was for the first time by way of separate


electorates was introduced by
a) Government of India Act of 1858
b) Minto- Morley Reforms of 1909
c) Indian Councils Act of 1892
d) Queen Victorias Proclamation

Solution: b)
Leaning: The Minto- Morley reforms never desired to set up a parliamentary
form of government in India.

The principle of election to the councils was legally recognized. But


communal representation was for the first time introduced in the
interests of Muslims. Separate electorates were provided for the
Muslims.

The number of members in provincial legislative councils of major


provinces was raised to 50.

The Councils were given right to discuss and pass resolutions on the
Budget and on all matters of public interest. However, the Governor-
General had the power to disallow discussion on the budget.

An Indian member was appointed for the first time to the Governor-
Generals Executive Council. Sir S. P. Sinha wasthe first Indian to be
appointed thus.

Q Source: 12th TN History Textbook

24. Consider the following about Saharia tribes.


www.insightsonindia.com
www.insightsias.com
TEST 28 Solutions

1. They are migratory tribes and originally belong to West Asia.


2. They can be found living in Central India.
3. They do not follow any religion and worship only natural forces.

Select the correct answer using the codes below.

a) 1 and 2 only
b) 2 and 3 only
c) 3 only
d) 2 only

Solution: d)
Justification: Statement 1 and 2: Sahariya is an indigenous Munda-
speaking tribe in the Madhya Pradesh province of India. The Saharias are
mainly found in the districts of Morena, Sheopur, Bhind, Gwalior, Datia,
Shivpuri, Vidisha and Guna districts of Madhya Pradesh and Baran district of
Rajasthan.

They are a Particularly Vulnerable Tribal Groups (PTGS).

Statement 3: Some sources indicate that they take part in Hindu Festivals,
and worship Hindu Gods and Goddesses.

Q Source:
http://tribal.nic.in/WriteReadData/CMS/Documents/2013060302040391137
51StatewisePTGsList.pdf

25.The Union environment ministry has banned the manufacture and


import of polychlorinated biphenyls (PCBs) and directed complete
prohibition on its use in any form by the end of 2025. Why is there
concern about the use of PCBs?
1. PCBs are persistent organic pollutants (POPs).
2. PCBs are highly inflammable and volatile.
3. PCBs are carcinogens.

Which of the above is/are correct?

a) 1 and 2 only
b) 2 and 3 only
www.insightsonindia.com
www.insightsias.com
TEST 28 Solutions

c) 1 only
d) 1 and 3 only

Solution: d)
Justification: The Regulation of Polychlorinated Biphenyls Order, 2016
bans import of equipment containing PCBs.

What are PCBs?

PCBs are synthetic organic chemicals that were initially accepted as an


important industrial breakthrough for various usages, but were later
found to be highly toxic, leading to demands for a ban on it or reducing
its use.
They were used in the manufacture of electrical equipment, heat
exchangers and other materials.

Issues

The governments decision is in line with the Stockholm Convention,


under which signatory countries are to prohibit or take necessary legal
and administrative measures to eliminate the production and use of
PCBs. India signed it in 2002.
PCBs are persistent organic pollutants (POPs) and are recognised by the
International Agency for Research on Cancer (IARC) as Group 1
carcinogens or cancer-causing substances.
Studies have also found that exposure to PCBs leads to serious health
problemsacute and chroniclike skin rashes, skin and fingernail
pigmentation changes, disturbances in liver function and the immune
system, irritation of the respiratory tract, depression, memory loss,
nervousness, impotence, elevated risk of cardiovascular disease,
hypertension and diabetes, liver damage and others.
As per estimates, 10,000 tonnes of PCBs have been recorded in India,
especially in the power sector. Though India never manufactured it but
we have stockpiles of it.

Q Source: MoEF Website (Orders section)


www.insightsonindia.com
www.insightsias.com
TEST 28 Solutions

26. Which of the following is NOT a Non-Banking Financial Company


(NBFC)?
a) Chit funds
b) Companies engaging in Insurance business
c) Companies engaging in stock broking
d) Self-Help Groups (SHGs) that raise loans from cooperatives

Solution: d)
Justification: India has financial institutions which are not banks but which
accept deposits and extend credit like banks. These are called Non-Banking
Financial Companies (NBFCs) in India.

So, clearly (d) is not a NBFC, since it doesnt extend credit or accept deposits.

Learning: NBFCs in India include not just the finance companies that the
general public is largely familiar with; the term also entails wider group of
companies that are engaged in investment business, insurance, chit fund,
nidhi, merchant banking, stock broking, alternative investments, etc., as their
principal business.

All are though not under the regulatory purview of the Reserve Bank.

Q Source: RBI Website

27. Consider the following statements about the Deccan plateau of


Peninsular India.
1. The eastern Deccan plateau is made of vast sheets of thick limestone
rocks.
2. The north-western part of the plateau is made up of lava flows.

Which of the above is/are correct?

a) 1 only
b) 2 only
c) Both 1 and 2
d) None

Solution: b)
www.insightsonindia.com
www.insightsias.com
TEST 28 Solutions

Justification: Statement 1: It is made of large sheets of granite. It effectively


traps rainwater. Under the thin surface layer of soil is the impervious gray
granite bedrock.

Statement 2: The rocks are spread over the whole of Maharashtra and parts of
Gujarat and Madhya Pradesh, thereby making it one of the largest volcanic
provinces in the world. The thick dark soil (called regur) found here is suitable
for cotton cultivation.

Q Source: Improvisation: Past year UPSC papers

28. In the Lok Sabha, who decides the sitting order of the members?
a) Member can sit at a place of their preference barring the first rows.
b) Secretary General, Lok Sabha
c) Leader of the House
d) Speaker, Lok Sabha

Solution: d)
Learning: These rules pertain to Lok Sabhas SUMMONS TO MEMBERS,
SEATING, OATH OR AFFIRMATION AND ROLL OF MEMBERS:

The Secretary-General shall issue summons to each member specifying the


date and place for a session of the House.

The members shall sit in such order as the Speaker may determine.

You should see important rules here at the Q Source.

Q Source: http://164.100.47.192/loksabha/rules.aspx

29. Which of the following are the benefits of Agro-forestry?


1. Reduced organic matter on the field and soil
2. More efficient recycling of nutrients by deep rooted trees on the site
3. Reduction of surface run-off and nutrient leaching

Select the correct answer using the codes below.

a) 1 and 2 only
www.insightsonindia.com
www.insightsias.com
TEST 28 Solutions

b) 3 only
c) 2 and 3 only
d) 1 and 3 only

Solution: c)
Justification: Agroforestry is a land use management system in which trees
or shrubs are grown around or among crops or pastureland.

Environmental benefits

Reduction of pressure on natural forests.

More efficient recycling of nutrients by deep rooted trees on the site as


crop roots do not reach as deep in the soil as that of trees, and often the
top soil is bereft of nutrients.

Better protection of ecological systems

Reduction of surface run-off, nutrient leaching and soil erosion through


impeding effect of tree roots and stems on these processes

Improvement of microclimate, such as lowering of soil surface


temperature and reduction of evaporation of soil moisture through a
combination of mulching and shading

Increment in soil nutrients through addition and decomposition of


litterfall.

Improvement of soil structure through the constant addition of organic


matter from decomposed litter. So, 3 is incorrect.

Q Source: Agritech TNAU Website

30. Excessive instream sand-and-gravel mining affects


1. Bank erosion and causes degradation of rivers
2. Adjoining groundwater system
3. Aquatic and riparian habitats

Select the correct answer using the codes below.

a) 1 only
www.insightsonindia.com
www.insightsias.com
TEST 28 Solutions

b) 2 and 3 only
c) 3 only
d) 1, 2 and 3

Solution: d)
Justification: Statement 1: Instream mining lowers the stream bottom,
which may lead to bank erosion. Depletion of sand in the streambed and along
coastal areas causes the deepening of rivers and estuaries, and the
enlargement of river mouths and coastal inlets. Any volume of sand exported
from streambeds and coastal areas is a loss to the system.

Statement 2: It may also lead to saline-water intrusion from the nearby sea.
The effect of mining is compounded by the effect of sea level rise. So, Sand
mining also affects the adjoining groundwater system and the uses that local
people make of the river.

Statement 3: Instream sand mining results in the destruction of aquatic and


riparian habitat through large changes in the channel morphology. Impacts
include bed degradation, bed coarsening, lowered water tables near the
streambed, and channel instability.

These physical impacts cause degradation of riparian and aquatic biota and
may lead to the undermining of bridges and other structures. Continued
extraction may also cause the entire streambed to degrade to the depth of
excavation.

Q Source: Often in news due to illegal Sand mining in UP

31. Consider the following statements about the Mahalwari system.


1. It eliminated middlemen between the government and the village
community.
2. It was introduced all over British India.
3. Every peasant was held personally responsible for direct payment of
land revenue to the government.

Select the correct answer using the codes below.

a) 1 only
b) 2 and 3 only
c) 1 and 3 only
d) 1, 2 and 3
www.insightsonindia.com
www.insightsias.com
TEST 28 Solutions

Solution: a)
Justification: Statement 1: Under this system the basic unit of revenue
settlement was the village or the Mahal. As the village lands belonged jointly to
the village community, the responsibility of paying the revenue rested with the
entire Mahal or the village community. So the entire land of the village was
measured at the time of fixing the revenue.

Statement 2: In 1833, the Mahalwari settlement was introduced in the


Punjab, the Central Provinces and parts of North Western Provinces.

Statement 3: It was the mahal (village), not the peasant who was responsible
for settling payments. This was a feature of Ryotwari, not Mahalwari.

Q Source: 12th TN History Textbook

32.Gandhiji decided to launch Individual Satyagraha due to


dissatisfaction with
a) Government of India Act, 1935 which eroded autonomy of local bodies
b) Religious intolerance in the countryside
c) August offer made by the British during the course of the Second World
War
d) State of untouchables in the country

Solution: c)
Learning: During the course of the Second World War in order to secure the
cooperation of the Indians, the British Government made an announcement
on August 1940, which came to be known as the August Offer.

The August Offer envisaged that after the War a representative body of
Indians would be set up to frame the new Constitution. Gandhi was not
satisfied with is offer and decided to launch Individual Satyagraha.

Individual Satyagraha was limited, symbolic and non-violent in nature


and it was left to Mahatma Gandhi to choose the Satyagrahis.

Acharya Vinoba Bhave was the first to offer Satyagraha and he was
sentenced to three months imprisonment.
www.insightsonindia.com
www.insightsias.com
TEST 28 Solutions

Jawaharlal Nehru was the second Satyagrahi and imprisoned for four
months. The individual Satyagraha continued for nearly 15 months.

Q Source: 12th TN History Textbook

33.The title Socrates of South Asia was given by UNESCO to


a) M. K. Gandhi
b) Aurobindo Ghosh
c) Deen Dayal Upadhyaya
d) E.V. Ramaswamy

Solution: d)
Learning: E.V. Ramasamy was an atheist and a bitter critic of the Vedas, but
his life was search for the truth, just like Socrates.

He led thousands of men and women in street processions to parody Hindu


gods and goddesses, and taught against superstition, blind belief and
dominance.

UNESCO awarded him with an unprecedented citation: The prophet of the


new age, the Socrates of South Asia, father of social reform movement and
arch enemy of ignorance, superstitions, meaningless customs and base
manners.

Q Source: 12th TN History Textbook

34. In an election to a State Legislative Assembly, all the candidates,


who did not win, lost their deposit. How is this possible?
a) The voter turnout was very low.
b) The winning member must have flouted Election Commissions
guidelines.
c) The election was for a multi-member constituency.
d) A very large number of candidates contested the election.
www.insightsonindia.com
www.insightsias.com
TEST 28 Solutions

Solution: d)
Justification: As per R. P. Act 1951, a general candidate for contesting an
Assembly election will have to make a security deposit of Rs. 10,000/-.

Deposit amount if lower for SC and ST candidates.

The deposit made by a candidate shall be returned if the following conditions


are satisfied:-

The candidate is not shown in the list of contesting candidates, that is to


say, either his nomination was rejected or after his nomination was
accepted, he withdrew his candidature; or
He dies before the commencement of the poll; or
He is elected; or
He is not elected but gets more than 1/6th of the total no.of valid votes
polled by all the candidates at the election.

However, If the candidate has polled exactly 1/6th of the total number of valid
votes polled by all the candidates, the deposit will not be refunded.

If the candidate was elected, the deposit will be refunded even if he did not
poll more than 1/6th of the total valid votes polled by all the candidates.

Q Source: ECI Website and past year UPSC papers

35. Tribal groups are at different stages of social, economic and educational
development. Particularly Vulnerable Tribal Groups (PVTGs) being one
of them. Which of the following is NOT a necessary characteristic of a
PVTG?
a) Stagnant or declining population
b) Pre-agriculture level of technology
c) Absence of social hierarchies
d) Extremely low literacy

Solution: c)
Learning: Tribal communities live, in various ecological and geo-climatic
conditions ranging from plains and forests to hills and inaccessible areas.
www.insightsonindia.com
www.insightsias.com
TEST 28 Solutions

While some tribal communities have adopted a mainstream way of life, at the
other end of the spectrum, there are certain Scheduled Tribes, 75 in number
known as Particularly Vulnerable Tribal Groups (PVTGs), who are
characterised by:- a) pre-agriculture level of technology; b) stagnant or
declining population; c) extremely low literacy; and d) subsistence level of
economy.

This test covers some of these tribes like Jarawas, Saharia etc.

Q Source: Ministry of Tribal Affairs website

36. Kisan Suvidha and Pusa Krishi for benefit of farmers are
a) Mobile applications
b) Agriculture Extension Centres
c) Soil Health cards
d) Government sponsored GM seed centres

Solution: a)
Learning: Ministry of Agriculture & Farmers Welfare has launched two
mobile apps called Kisan Suvidha and Pusa Krishi for benefit of farmers and
all other stake holders.

Pusha Krishi app will provide farmers with information related to new
varieties of crops developed by Indian Council of Agriculture Research (ICAR),
resource conserving cultivation practices as well as farm machinery and its
implementation will help in increasing returns to farmers.

Both have been developed in house.

Q Source: http://agricoop.nic.in/

37. Which of the following is NOT a fundamental duty under Part IVA of the
Constitution?
a) To pay taxes on time
b) To respect the National Flag
www.insightsonindia.com
www.insightsias.com
TEST 28 Solutions

c) To protect the environment


d) To develop a scientific and rational temper

Solution: a)
Learning: Option (a) was recommended by Swaran Singh Committee by was
not accepted by the Government.

Option (b): To abide by the Constitution and respect its ideals and institutions,
the National Flag and the National Anthem.

Option (c): To protect and improve the natural environment including forests,
lakes, rivers and wildlife and to have compassion for living creatures

Option (d): To develop scientific temper, humanism and the spirit of inquiry
and reform.

Q Source: Indian Polity: M Laxmikanth

38. The Indian Parliamentary Group is


a) An autonomous body, membership of which is open to all current
or former members of the Indian Parliament.
b) A standing committee of the Parliament presided by the Speaker
c) A youth forum eliciting their participation in active national
politics
d) An attached office under the Ministry of Parliamentary Affairs
that trains and assists the newly elected MPs in discharge of their
responsibilities

Solution: a)
Learning: The Indian Parliamentary Group is an autonomous body formed
in the year 1949 in pursuance of a Motion adopted by the Constituent
Assembly (Legislative) in 1948.

Membership of the Indian Parliamentary Group is open to all Members of


Parliament and ex-Members of Parliament.

A Member of Parliament can become a life Member of the Group on payment


of life subscription of Rs. 500/- by sending an application together with the
www.insightsonindia.com
www.insightsias.com
TEST 28 Solutions

life subscription to the Secretary-General of Lok Sabha (who is ex-officio


Secretary-General of the Group).

The aims and objects of the Indian Parliamentary Group are:

to promote personal contacts between Members of Parliament;

to study questions of public importance that are likely to come up before


Parliament and arrange Seminars and discussions and orientation
courses and bring out publications for the dissemination of information
to the Members of the Indian Parliamentary Group;

to arrange lectures on political, defence, economic, social and


educational problems by Members of Parliament and distinguished
persons; and

to arrange visits to foreign countries with a view to develop contacts


with Members of other Parliaments.

Q Source:
http://164.100.47.192/loksabha/writereaddata/our%20parliament/Indian%2
0Parliamentary%20Group.pdf

39. Some of the major coal belts of India in Jharia and Raniganj are
affected by mine fires and coal subsidence. According to official sources,
what is the primary cause of this?
a) Un-scientific and illegal mining leading to self-combustion of coal
b) Geologically young coal formations in these regions
c) Global Warming and climate change
d) High price of coal in these regions

Solution: a)
Learning: Jharia is famous for a coal field fire that has burned underground
for nearly a century. The first fire was detected in 1916. Flames at Jharia have
been recorded reaching heights of 60 feet.
www.insightsonindia.com
www.insightsias.com
TEST 28 Solutions

Many of the fires are believed to have started through spontaneous


combustion or what is sometimes called self-heating, caused by the oxidation
of minerals in the exposed coal.

Much of the mining at Jharia is done at the surface, exposing the coal to open
air. Certain varieties of coal contain minerals that heat up when they are
exposed to oxygen; those minerals can in turn burst into flames.

The population living in the old mining areas has increased many times over
the years, though these areas became unsafe for habitation. Presently the
Jharkhand state government is running a resettlement program for the region.

Q Source:
http://coal.gov.in/sites/upload_files/coal/files/coalupload/Chap9AnRep1314
.pdf

40. Off-grid Renewable Energy can be used to meet the energy


requirements of isolated communities and areas which are not likely to
be electrified in near future. Which of the following can NOT be used as
a part of such systems?
a) Biomass gasifiers
b) Micro hydro projects
c) Solar Photo Voltaic Roof-top Systems
d) All can be used as part of such systems.

Solution: d)
Justification & Learning: Distributed/decentralized renewable power
projects using wind energy, biomass energy, hydro power and hybrid systems
are being established in the country to meet the energy requirements of
isolated communities and areas which are not likely to be electrified in near
future.

Off-grid Renewable Energy / Power can be generated using the following:

Biomass based heat and power projects and industrial waste to-energy
projects for meeting captive needs
Biomass gasifiers for rural and industrial energy applications
www.insightsonindia.com
www.insightsias.com
TEST 28 Solutions

Watermills/micro hydro projects for meeting electricity requirement


of remote villages
Small Wind Energy & Hybrid Systems - for mechanical and electrical
applications, mainly where grid electricity is not available.
Solar PV Roof-top Systems for abatement of diesel for power generation
in urban areas

The Ministry of New and Renewable energy is running a program with main
objectives of: supporting RD&D to make such systems more reliable and cost-
effective, demonstration, field testing, strengthening manufacturing base.

Q Source: Ministry of New and Renewable energy website

41. Consider the following statements.


1. They are cut into the volcanic lava of the Deccan in the forest ravines
of the Sahyadri Hills.
2. These caves contain carvings that depict the life of Buddha.
3. The carvings and sculptures in the cave are considered to be the
beginning of classical Indian art.

The above refers to?

a) Ajanta caves
b) Barabar caves
c) Nural caves
d) Kailasha Caves

Solution: a)
Justification: The question seems to be very factual on first glance, but
elimination can easily give the correct answer.

Barabar caves are in Bihar. So, (b) cant be the answer, as Sahyadri hills do not
pass through Bihar.
www.insightsonindia.com
www.insightsias.com
TEST 28 Solutions

Nural caves is a made up name, and Varaha Cave, as also clear from the name,
hosts sculptures of Lord Vishnu. So, the only possible answer is (a), which is
also quite renowned.

Learning: The style of Ajanta has exerted a considerable influence in India


and elsewhere, extending, in particular, to Java.

The caves include paintings and sculptures described by the government


Archaeological Survey of India as "the finest surviving examples of Indian art,
particularly painting", which are masterpieces of Buddhist religious art, with
figures of the Buddha and depictions of the Jataka tales.

The caves were built in two phases starting around the 2nd century BCE, with
the second group of caves built around 400650 CE.

The site is a protected monument in the care of the Archaeological Survey of


India, and since 1983, the Ajanta Caves have been a UNESCO World Heritage
Site.

Q Source: http://www.indiaculture.nic.in/ajanta-caves

42. Which of the following is NOT one of the sessions of Parliament?


a) Budget Session
b) Economy Session
c) Winter Session
d) Monsoon Session

Solution: b)
Learning: The period during which the parliamentarians convene and conduct
the business of each House is referred to as session. Every year, the Parliament
of India conducts three sessions Budget, Monsoon and Winter.

Budget session, which is conducted from February to May, is considered


highly crucial for the matters it deals with.

Monsoon session: After a brief hiatus of around two months, both the Houses
of the Parliament resume business in July and continue till September. Unlike
www.insightsonindia.com
www.insightsias.com
TEST 28 Solutions

the preceding session, the monsoon session is dedicated largely to discussions


on matters of public interest. It is during the monsoon session that the
members of the Parliament, including the ministers, can bring forward the
legislative proposals in the form of a Bill

Winter Session: It is the shortest session that spans over a period of one
month generally between mid-November and mid-December. It takes up the
issues that could not be deliberated upon and makes up for the lack of
legislative business during the second session of the Parliament.

Q Source: Indian Polity: M Laxmikanth

43. Which of the following describe most accurately the character of


precipitation in India?
a) Rainfall is dispersed equally over the year.
b) Most of the rainfall is concentrated in few months of time.
c) Rainfall period is highly uneven in India, with sudden spurts
occurring at non-definite times.
d) There is no clear pattern of precipitation in India.

Solution: b)
Learning: Monsoonal showers supply over 80% of India's annual rainfall.

Some of the states that do not receive this rainfall get rainfall by retreating
Monsoon, like TN, parts of AP.

Some states in the North-east see rainfall for most of the year.

But, the larger pattern of precipitation in India is monsoonal.

Q Source: 9th Geography NCERT


www.insightsonindia.com
www.insightsias.com
TEST 28 Solutions

44. Highest total replenishable ground water resources lie in which


river basin?
a) Pennar
b) Krishna
c) Ganga
d) Godavari

Solution: c)
Justification: Highest is in Ganga, followed by Govarai, Brahmaputra,
Indus, Krishna.

Gangas groundwater resources are at least 4-5 times the resources in these
rivers.

You can see the image below from www.nih.ernet.in

Q Source:
http://www.nih.ernet.in/rbis/india_information/groundwater.htm
www.insightsonindia.com
www.insightsias.com
TEST 28 Solutions

45.Which of the following ministries/departments has the key role in the


Governance of SEZs in India?
a) Ministry of External Affairs
b) Department of Commerce
c) Department of Economic Affairs
d) Ministry of Heavy Industries and Enterprises

Solution: b)
Justification: Option (b) is verified by the following:

In the SEZ board of approval, Secy, Dept. of Commerce is the Chairman.


There is an office of Development Commissioner of Special Economic
Zones (SEZs).
Director General of Foreign Trade is under Ministry of Commerce and
Industry.

Learning: For getting approval for setting SEZs, The developer submits the
proposal for establishment of SEZ to the concerned State Government.

The State Government has to forward the proposal with its recommendation
within 45 days from the date of receipt of such proposal to the Board of
Approval.

The applicant also has the option to submit the proposal directly to the Board
of Approval.

The Board of Approval has been constituted by the Central Government in


exercise of the powers conferred under the SEZ Act.

All the decisions are taken in the Board of Approval by consensus. The Board
of Approval has 19 Members including Secy. from key ministries.

Q Source: http://commerce.nic.in/DOC/InnerContent.aspx?Id=216
http://sezindia.gov.in/

46. Which of the following prevent the establishment of an


authoritarian and despotic rule in the country, operate as limitations on
www.insightsonindia.com
www.insightsias.com
TEST 28 Solutions

the arbitrary laws of the legislature and protect the liberties of the
people against the invasion by the State?
a) Directive Principles of State Policy
b) Fundamental Duties
c) Federal Structure of India
d) Fundamental Rights enshrined in the Constitution

Solution: d)
Learning: The Fundamental Rights are meant for promoting the ideal of
political democracy. They operate as limitations on the tyranny of the
executive and arbitrary laws of the legislature. In short, they aim at
establishing a government of laws and not of men.

The Fundamental Rights are named so because they are guaranteed and
protected by the Constitution, which is the fundamental law of the land. They
are fundamental also in the sense that they are most essential for the all-
round development (material, intellectual, moral and spiritual) of the
individuals.

Q Source: Indian Polity: M Laxmikanth

47. Land degradation has far-reaching consequences leading to


1. Decline in water table
2. Reduced agricultural productivity
3. Loss of bio-diversity

Select the correct answer using the codes below.

a) 2 only
b) 1 and 3 only
c) 1 and 2 only
d) 1, 2 and 3

Solution: d)
Justification: Statement 1: Loss of vegetation leads to soil losing ability to
retain large amounts of water (so that they slowly seep through to the water
table), resulting in reduced water table.
www.insightsonindia.com
www.insightsias.com
TEST 28 Solutions

Statement 2: Loss of top soil, nutrients and organic matter due to degradation
reduces productivity.

Statement 3: Deforestation, loss of top soil etc cause large scale killing of
beneficial microorganisms, earthworms, forces large species to shift habitats,
damages local ecosystems etc.

All these affect the lives and livelihoods of the populations, often eventually
precipitating forced migration and socio-economic conflicts.

Q Source: MoEF Website

48. Consider the following about the Science and Engineering


Research Board (SERB).
1. It is a statutory body established by the Parliament
2. It is mandated to fund grass-root innovations in rural areas.

Which of the above is/are correct?

a) 1 only
b) 2 only
c) Both 1 and 2
d) None

Solution: a)
Justification: Statement 1: It was setup in 2008 by the Parliament.

Statement 2: It supports basic research in emerging areas of Science &


Engineering. It serves as a premier agency for planning, promoting and
funding of internationally competitive research in emerging areas.

It also works to:

Identify major inter-disciplinary research areas, and individuals, groups


or institutions and funding them for undertaking research.
Assist in setting up infrastructure and environment for scientific
pursuit.
www.insightsonindia.com
www.insightsias.com
TEST 28 Solutions

Enable in order to achieve synergy between academic institutions,


research and development laboratories and industry for promoting
basic research in science and engineering.
Evolve a system of approach to expeditiously provide funding for
research, including monitoring and evaluation, by adopting modern
management practices.

Q Source: http://dst.gov.in/statutory-board-0

49. Rotterdam Convention deals with


a) Greenhouse gases
b) Hazardous Chemicals in International Trade
c) Preservation of genetic resources
d) Ozone depletion

Solution: b)
Learning: India has ratified the convention.

Rotterdam Convention was adopted in 1998 by a Conference of


Plenipotentiaries in Rotterdam, the Netherlands. The Convention entered into
force in 2004.

The objectives of the Convention are:

to promote shared responsibility and cooperative efforts among Parties


in the international trade of certain hazardous chemicals in order to
protect human health and the environment from potential harm;

to contribute to the environmentally sound use of those hazardous


chemicals, by facilitating information exchange about their
characteristics, by providing for a national decision-making process on
their import and export and by disseminating these decisions to Parties.

The Convention creates legally binding obligations for the implementation of


the Prior Informed Consent (PIC) procedure. It built on the voluntary PIC
procedure, initiated by UNEP and FAO in 1989 and ceased on 24 February
2006.
www.insightsonindia.com
www.insightsias.com
TEST 28 Solutions

Q Source: MoEF Website

50. Cryptography is related to the practice of techniques for


a) Secure communication in the presence of third parties
b) Connecting local network with major networks
c) Dealing with e-wastes
d) Mobile-governance that has the potential to transform
government communication

Solution: a)
Solution: Cryptography is about constructing and analyzing protocols that
prevent third parties or the public from reading private messages.

Various aspects in information security such as data confidentiality, data


integrity, authentication, and non-repudiation are central to modern
cryptography.

Modern cryptography exists at the intersection of the disciplines of


mathematics, computer science, and electrical engineering. Applications of
cryptography include ATM cards, computer passwords, and electronic
commerce.

Q Source: DeITY Website

51. National Coastal Zone Management Authority consists of the following


persons
1. Minister of Environment, Forests and Climate Change as Chairman
2. Minister of State for Water Resources
3. State Chief Ministers

Select the correct answer using the codes below.

a) 1 only
www.insightsonindia.com
www.insightsias.com
TEST 28 Solutions

b) 2 and 3 only
c) 2 only
d) None of the above

Solution: d)
Justification: NCZMA consists of:

Secretary, Ministry of Environment and Forests, New Delhi - Chairman


Director, National Institute of Oceanography, Goa - Member
Chief Town Planner, Town and Country Planning Organization,
Government of Goa - Member
Member or an Officer of an equivalent rank, Central Groundwater
Board, New Delhi - Member
Joint Secretary (Tourism), or his representative, Ministry of Tourism,
New Delhi - Member
Director General (Fisheries), Ministry of Agriculture, New Delhi -
Member
Other members (please see Q Source for this)

Learning: It was established by an order of the executive. The Authority shall


have the power to take the following measures for protecting and improving
the quality of the coastal environment and preventing, adapting and
controlling environmental pollution in coastal areas, namely:

Co-ordination of actions by the State Coastal Zone Management


Authorities and the Union territory Coastal Zone Management
Authorities under the said Act and the rules made there under, or under
any other law which is relatable to the objects of the said Act.
Examination of the proposals for changes and modifications in
classification of coastal zone areas and in the Coastal Zone Management
Authorities and making specific recommendations to the Central
Government therefore;
Review of cases involving violations of the provisions of the said Act and
the rules

Q Source:
http://www.moef.nic.in/sites/default/files/Minutes%20of%20Meeting%2030
.PDF
www.insightsonindia.com
www.insightsias.com
TEST 28 Solutions

52.India has improved its ranking in the World Bank Group's bi-annual
"Logistics Performance Index 2016", jumping from 54th in 2014 to 35th
in 2016. The Logistics Performance Index is published by
a) WTO
b) World Bank
c) UNECOSOC
d) IMF

Solution: b)
Learning: Global trade depends on logistics and how efficiently countries
import and export goods determine how they grow and compete in the global
economy.

Countries with efficient logistics can easily connect firms to domestic


and international markets through reliable supply chains.

Countries with inefficient logistics face high costs both in terms of


time and money in international trade and global supply chains.

The World Bank Group's bi-annual report Connecting to Compete


2016: Trade Logistics in the Global Economy, launched on Wednesday,
captures critical information about the complexity of international
trade.

The Logistics Performance Index (LPI) within the report scores 160
countries on key criteria of logistics performance.

Among the six sub-indices of the Logistics Performance Index, India


improved the most on "the efficiency of customs and border
management clearance"

Q Source: Ministry of Commerce Website Press Releases

53. Which of the following is NOT one of the Consumer Rights in India?
a) Right to Safe goods
b) Right to be informed about the quality of goods
c) Right to free public goods and services
d) Right to consumer education
www.insightsonindia.com
www.insightsias.com
TEST 28 Solutions

Solution: c)
Learning: The consumer has a Right to Choose, not right to free goods and
services.

It means right to be assured, wherever possible of access to variety of goods


and services at competitive price.

In case of monopolies, it means right to be assured of satisfactory quality and


service at a fair price. It also includes right to basic goods and services.

This is because unrestricted right of the minority to choose can mean a denial
for the majority of its fair share.

This right can be better exercised in a competitive market where a variety of


goods are available at competitive prices

Other rights are right to be heard, right to seek redressal etc.

Q Source: NCERT 10th Economics

54.Which of the following in India is NOT categorized as a Wetland?


a) Coastal Salt pans
b) Inter-tidal mud flat
c) Reservoir or Barrage
d) Wet grasslands

Solution: d)
Learning: See the image below, it gives details of all identified types of
Wetland in India. The survey was done by ISRO.

As per the definition of wetland, diverse types of classes get included like
lagoon, beach, mangrove, coral reef, salt pan, aquaculture pond, waterlogged,
ox-bow lake, reservoir, lake, tank inter-tidal mudflat etc.
www.insightsonindia.com
www.insightsias.com
TEST 28 Solutions

Q Source:
http://www.moef.nic.in/sites/default/files/NWIA_National_brochure.pdf

55. Consider the following about Mamallapuram Utsav.


1. It is celebrated in Tamil Nadu.
2. Indian classical dances and folk dances are performed in this festival.
3. It received the tag of Intangible UNESCO heritage recently.

Select the correct answer using the codes below.

a) 1 only
b) 1 and 2 only
c) 2 only
d) 1 and 3 only

Solution: b)
www.insightsonindia.com
www.insightsias.com
TEST 28 Solutions

Justification: Mahabalipuram, a small city in Tamil Nadu, is famous for its


temples that represent the architectural beauty of the ancient India.

It is also famous for its vibrant Mamallapuram Dance Festival. Indian classical
dances including Bharatanatyam, Kuchipudi, Kathak, Mohiniattam, Odissi
and Kathakali are performed in this festival, which is held annually, during the
months of January and February.

Apart from the classical dances, folk dances are also performed in the festival.

The cultural event is promoted by the Tamil Nadu Tourism Department. The
four-week dance festival has evolved as one of the major cultural attractions of
the southern state of India.

Q Source: Ministry of Culture Website

56.The GRIHA Council performs which of the following functions?


1. Monitoring housing shortages in India and coordinating with
Ministry of Urban Development to execute cooperative housing
projects
2. Resettlement of slum residents to in metropolitan areas

Which of the above is/are correct?

a) 1 only
b) 2 only
c) Both 1 and 2
d) None

Solution: d)
Justification: GRIHA is mandated to promote development of buildings and
habitats in India through GRIHA.

It is an independent platform for the interaction on scientific and


administrative issues related to sustainable habitats in the Indian
subcontinent, not execution of projects. So, 1 is wrong. Similarly, 2 is wrong,
as GRIHA is not an executive agency, but a think tank.
www.insightsonindia.com
www.insightsias.com
TEST 28 Solutions

Learning: It was founded by TERI (The Energy and Resources Institute, New
Delhi) with support from MNRE (Ministry of New and Renewable Energy,
Government of India) along with a handful of experts in the sustainability of
built environment from across the country.

It conducts awareness workshops on Green Buildings and GRIHA rating


system to all the registered project teams.

Q Source: Ministry of New and Renewable energy website

57. Which of the following administrative reforms is/are associated with


Cornwallis, a Governor-General of British India?
1. Making appointments mainly on the basis of merit thereby laying the
foundation of the Indian Civil Service
2. Separation of the commercial, judicial and revenue branch of
services

Which of the above is/are correct?

a) 1 only
b) 2 only
c) Both 1 and 2
d) None

Solution: c)
Justification: Statement 1: The greatest work of Cornwallis was the
purification of the civil service by the employment of capable and honest
public servants.

Cornwallis, who aimed at cleansing the administration, abolished the


vicious system of paying small salaries and allowing enormous
perquisites.

He persuaded the Directors of the Company to pay handsome salaries to


the Company servants in order that they might free themselves from
commercial and corrupting activities.

Further, Cornwallis inaugurated the policy of making appointments


mainly on the basis of merit thereby laying the foundation of the Indian
Civil Service.
www.insightsonindia.com
www.insightsias.com
TEST 28 Solutions

Statement 2: A major reform that Cornwallis introduced was the separation of


the three branches of service, namely commercial, judicial and revenue. The
collectors, the king-pins of the administrative system were deprived of their
judicial powers and their work became merely the collection of revenue.

Also, Civil and criminal courts were completely reorganized.

Q Source: 12th TN History Textbook

58. Which of the following were the Main features of the Permanent
Settlement?
1. The zamindars of Bengal were recognised as the owners of land but
Government had a right to take away the land.
2. The ryots became tenants since they were considered the tillers of the
soil.
3. It took away the administrative and judicial functions of the
zamindar.
4. Revenue was fixed on an annual lease basis but revisable at the end
of every financial year.

Select the correct answer using the codes below.

a) 1, 2 and 3 only
b) 2 and 4 only
c) 1 and 3 only
d) 1, 2, 3 and 4

Solution: a)
Justification: Statement 1: They were recognised as the owners of land as
long as they paid the revenue to the East India Company regularly.

Statement 2: Since zamindars became owners, ryots who cultivated the fields
now started working as Tenants.

Statement 3: Zamindars lost the control over land, revenue settling powers etc
and ultimately became pawns in the hands the British.

Statement 4: The amount of revenue that the zamindars had to pay to the
Company was firmly fixed and would not be raised under any circumstances.
www.insightsonindia.com
www.insightsias.com
TEST 28 Solutions

Q Source: 12th TN History Textbook

59.The IRNSS will provide services for


1. Standard Positioning Service (SPS) which can be used by Civilians
2. Restricted Service (RS) which can be used only by authorized sources
3. Encrypted Service (ES) which can be accessed only from space
observatories

Select the correct answer using the codes below.

a) 1 only
b) 1 and 3 only
c) 2 only
d) 1 and 2 only

Solution: d)
Learning: IRNSS space Segment consists of seven satellites, three satellites
in GEO stationary orbit (GEO) and four satellites in Geo Synchronous Orbit
(GSO) orbit.

The classification of services Standard and Restricted is done to secure


navigation data that is used by military, intelligence agencies, government etc.

Other standard data will be available to general public just like GPS. So, both 1
and 2 are correct.

There is no such service named as Encryption Service (ES) that will be


provided by IRNSS. So, 3 is incorrect.

Learning: IRNSS is an independent regional navigation satellite system


being developed by India. It is designed to provide accurate position
information service to users in India as well as the region extending up to 1500
km from its boundary, which is its primary service area.

Q Source: ISRO Website


www.insightsonindia.com
www.insightsias.com
TEST 28 Solutions

60. Small and payments banks are niche banks that


a) Lend large sums of money to MNCs and only cooperative banks
b) Regulate local finance institutions
c) Raise funding for Micro-finance Institutions operating in rural
areas
d) None of the above

Solution: d)
Learning: Small banks primarily undertake basic banking activities of
acceptance of deposits and lending to small farmers, businesses, MSMEs and
unorganized sector entities.

Payment bank is restricted to acceptance of demand deposits and provision of


payments and remittances. It cannot lend money to people like small banks.

So, none of the options (a), (b) or (c).

They were setup based on Nachiket Mor panel on financial inclusion.

Q Source: In news

61. The Self-respect Movement of the 20th Century was


1. Started from Madras Presidency.
2. Associated with E. V. Ramasamy.
3. Aimed at removing untouchability.

Select the correct answer using the codes below.

a) 2 only
b) 1 and 3 only
c) 2 and 3 only
d) 1, 2 and 3

Solution: d)
www.insightsonindia.com
www.insightsias.com
TEST 28 Solutions

Justification: Abolition of untouchability became a major issue of the 19th


and 20th century social and religious reform movements in the country.

The Self-Respect Movement was aimed at achieving a society where backward


castes have equal rights, and encouraged backward castes to have self-respect
in the context of a caste-based society that considered them to be a lower end
of the hierarchy.

It was founded in 1925 by E. V. Ramasamy (also called as Periyar by his


devoted followers) in Tamil Nadu, India against Brahminism.

The movement was extremely influential not just in Tamil Nadu, but also
overseas in countries with large Tamil populations, such as Malaysia and
Singapore.

Q Source: 12th TN History Textbook

62. The main objective of the Public Liability Insurance Act 1991 is to
a) Provide for damages to victims of an accident which occurs as a
result of handling any hazardous substance
b) Improve insurance penetration in India to 100%
c) Work towards achieving zero operator liability in case of nuclear
accidents
d) Reduce government financial liability and fiscal deficit to
sustainable levels

Solution: a)
Learning: With the growth of hazardous industries, risks from accidents
processes and operations, not only to the persons employed in such
undertakings but also to the public who may be in vicinity, have increased.

While the workers and employees of hazardous installations are protected


under separate laws, members of the public are not assured of any relief
except through long legal process. To ameliorate the sufferings of members of
the public due to accidents which take place in hazardous installations it was
found essential to provide for a Public Liability Insurance.
www.insightsonindia.com
www.insightsias.com
TEST 28 Solutions

The Act applies to all owners associated with the production or handling of
any hazardous chemicals.

It is important with regard to Bhopal Gas tragedy like incidents, nuclear


accidents (which were recently in news due to Indias civil nuclear liability
law) etc.

Q Source: http://envfor.nic.in/rules-regulations/public-liability-insurance

63. The development of LVM3 by ISRO will be significant because


a) It will allow India to establish a permanent space station for
scientific pursuits.
b) It will allow India to achieve complete self reliance in launching
geosynchronous satellites.
c) It will allow India to launch satellites from ocean territories
saving valuable land and space.
d) It will lead to development of more efficient missile launchers.

Solution: b)
Learning: LVM3 is a heavy launch capability launcher being developed by
ISRO.

It will allow India to achieve complete self reliance in launching


satellites as it will be capable of placing 4 tonne class Geosynchronous
satellites into orbit.

The LVM3 will have an India built cryogenic stage with higher capacity
than GSLV.

A cryogenic stage engine is more efficient and has a higher capacity than
normal engines, as it operates at extremely low temperatures.

The first experimental flight of LVM3, the LVM3-X/CARE mission lifted


off from Sriharikota in 2014 and successfully tested the atmospheric
phase of flight.
www.insightsonindia.com
www.insightsias.com
TEST 28 Solutions

Q Source: http://www.isro.gov.in/launchers/lvm3

64. Which of the following statements about Surendranath Banerjee


is INCORRECT?
a) He was in favour of partition of Bengal due to its administrative
advantages.
b) He had convened the Indian National Conference which merged
with the Indian National Congress.
c) He was the first Indian to become a Member of the British House
of Commons.
d) Both (a) and (c)

Solution: d)
Learning: Surendranath Banerjee was called the Indian Burke. He firmly
opposed the Partition of Bengal. So, (a) is wrong.

He founded the Indian Association (1876) to agitate for political reforms. He


had convened the Indian National Conference (1883) which merged with the
Indian National Congress in l886. So, (b) is correct.

Dadabhai Naoroji, who was regarded as Indias unofficial Ambassador in


England, was the first Indian to become a Member of the British House of
Commons. So, (c) is also incorrect.

Q Source: 12th TN History Textbook

65.Consider the following matches of Particularly Vulnerable Tribal Groups


(PTGS) with the State/UT they are found in.
1. Birhor : Bihar
www.insightsonindia.com
www.insightsias.com
TEST 28 Solutions

2. Totos : Rajasthan
3. Shom pens and Jarawa: Andaman and Nicobar Islands

Select the correct answer using the codes below.

a) 1 and 2 only
b) 1 and 3 only
c) 2 and 3 only
d) 1, 2 and 3

Solution: b)
Justification: Statement 1: A forest-dwelling society of about 4,000 people
in 2001, who live in central and eastern India in the states of Orissa,
Chhattisgarh, West Bengal, and Jharkhand with the greatest concentration in
the Ranchi, Hazaribagh, and Singhbhum districts of Jharkhand. Several recent
reports have indicated that the poulation may have dropped to 4,000 people.

Statement 2: The Toto is a primitive and isolated tribal group residing only in
a small enclave called Totopara in the Jalpaiguri of West Bengal, India.
Totopara is located at the foot of the Himalayas just to the south of the
borderline between Bhutan and West Bengal (on the western bank of Torsa
river). Read more here http://wbnorthbengaldev.gov.in/HtmlPage/toto.aspx

Statement 3: The Shom Pen live a nomadic life moving from one habitation to
another in search of fruit and hunt that is till now found abundantly and
sufficiently in the thick rain forests of the Great Nicobar Island. You can read
more here http://tribal.nic.in/Contents.aspx?mo=7&li=65

About Jarawas recently in news -


http://pib.nic.in/newsite/PrintRelease.aspx?relid=142204

Q Source:
http://tribal.nic.in/WriteReadData/CMS/Documents/2013060302040391137
51StatewisePTGsList.pdf

66. Some of the awards related to environment given by the


Government of India is/are NOT named after
1. Indira Gandhi
www.insightsonindia.com
www.insightsias.com
TEST 28 Solutions

2. Amrita Devi Bishnoi


3. Kailash Satyarthi

Select the correct answer using the codes below.

a) 1 and 2 only
b) 2 only
c) 3 only
d) 1, 2 and 3

Solution: c)
Justification: If you knew about Amrita Bishnoi Wildlife protection award,
the answer could have been reached easily.

Kailash Satyarthi, a nobel laureate, works for Child rights. So, 3 is incorrect,
and then the only answer can be (c).

Statement 1: This award is given for significant contribution in the field of


wildlife protection, which is recognised as having shown exemplary courage or
having done exemplary work for the protection of wildlife.

Statement 2: Shortlisting of nominations received for IGPP is carried out by


three Expert Members, selected by the Prime Ministers Office, out of a panel
of 9 eminent environmentalists/ persons prepared by the Ministry of
Environment, Forest and Climate Change.

Selection of awardees out of the shortlisted nominations is done by the


Environmental Prize Committee constituted under the Chairmanship of
Honble Vice President of India.

Q Source: http://www.moef.nic.in/

67. The Treaty of Sevres, 20th Century, is associated with which of the
following movements in British India?
a) Swaraj movement
b) Quit India movement
c) Khilafat movement
d) Civil Disobedience movement
www.insightsonindia.com
www.insightsias.com
TEST 28 Solutions

Solution: c)
Learning: The chief cause of the Khilafat Movement was the defeat of Turkey
in the First World War. The harsh terms of the Treaty of Sevres (1920) was felt
by the Muslims as a great insult to them.

The whole movement was based on the Muslim belief that the Caliph (the
Sultan of Turkey) was the religious head of the Muslims all over the world. The
Muslims in India were upset over the British attitude against Turkey and
launched the Khilafat Movement.

Q Source: 12th TN History Textbook

68. The Sustainable land and Ecosystem Management (SLEM)


Programme is a joint initiative of Government of India and
a) Global Environmental Facility (GEF)
b) UNESCO
c) World Wildlife Fund (WWF)
d) Conservation International

Solution: a)
Learning: The SLEM Programme is a joint initiative of the Government of
India and the Global Environmental Facility (GEF) under the latters Country
partnership Programme (CPP).

The objective of the SLEM Programmatic Approach is to promote sustainable


land management and use of biodiversity as well as maintain the capacity of
ecosystems to deliver goods and services while taking into account climate
change

It is a long-term and strategic arrangement of individual yet interlinked


projects aimed at achieving large-scale impacts on the global environment.

Q Source: MoEF Website


www.insightsonindia.com
www.insightsias.com
TEST 28 Solutions

69. The Poona Pact 1932 was signed to


a) Grant separate electorates to depressed classes
b) Implement the recommendations of the Second Round Table
Conference
c) Take back the Civil Disobedience movement
d) None of the above

Solution: d)
Learning: The Pact provided for reservations for depressed classes, not
separate electorates. So, (a) is wrong.

According to the Communal Award, the depressed classes were considered as


a separate community and as such provisions were made for separate
electorates for them.

Mahatma Gandhi protested against the Communal Award and went on a fast
unto death in the Yeravada jail on in 1932.

Finally, an agreement was reached between Dr Ambedkar and Gandhi. This


agreement came to be called as the Poona Pact. The British Government also
approved of it. Accordingly, 148 seats in different Provincial Legislatures were
reserved for the Depressed Classes in place of 71 as provided in the Communal
Award

Q Source: 12th TN History Textbook

70. C and Ku-bands often in news are related to


a) Communication satellites
b) Indo-China border skirmishes
c) Central Pay Commission
d) Air quality standards India

Solution: a)
Justification: The INSAT communication satellite system in the C,
Extended C and Ku-bands provides services to telecommunications, television
www.insightsonindia.com
www.insightsias.com
TEST 28 Solutions

broadcasting, satellite news gathering, societal applications, weather


forecasting, disaster warning and Search and Rescue operations.

Learning: The Indian National Satellite (INSAT) system is one of the largest
domestic communication satellite systems in Asia-Pacific region with nine
operational communication satellites placed in Geo-stationary orbit.

Established in 1983 with commissioning of INSAT-1B, it initiated a major


revolution in Indias communications sector and sustained the same later.
Currently operational communication satellites are INSAT-3A, INSAT-3C,
INSAT-3E, INSAT-4A, INSAT-4B, INSAT-4CR, GSAT-8, GSAT-10 and GSAT-
12.

Q Source: http://www.isro.gov.in/spacecraft/communication-satellites

71. Internet Domain Name System (DNS) is managed by


a) National governments
b) Internet Corporation for Assigned Names and Numbers (ICANN)
c) UN Commission on Internet Governance
d) All of the above jointly manage it

Solution: b)
Justification: To reach another person on the Internet you have to type an
address into your computer - a name or a number. That address has to be
unique so computers know where to find each other.

ICANN coordinates these unique identifiers across the world. Without that
coordination we wouldn't have one global Internet.

Learning: ICANN was formed in 1998. It is a not-for-profit partnership of


people from all over the world dedicated to keeping the Internet secure, stable
and interoperable. It promotes competition and develops policy on the
Internets unique identifiers.

ICANN doesnt control content on the Internet. It cannot stop spam and it
doesnt deal with access to the Internet.

But through its coordination role of the Internets naming system, it does have
an important impact on the expansion and evolution of the Internet.
www.insightsonindia.com
www.insightsias.com
TEST 28 Solutions

Q Source: DeITY Website

72. Tashkent Agreement 1966 intended to


a) Restore normal and peaceful relations between India and
Pakistan
b) Stop the Bangladeshi Liberation war
c) Allow India to import nuclear fuel from Central Asian countries
d) Settle boundaries of the North-Eastern Frontier between India
and China

Solution: a)
Justification: The question is easy if you eliminate the options.

India could not have needed nuclear fuel way back in 1966, so (c) is incorrect.

The Bangladeshi Liberation war happened in 1971, so (b) automatically


becomes wrong.

Learning:

Lal Bahadur Sastri ended the Indo-Pak war of 1965 by concluding the
Tashkent Agreement in January 1966.

Both nations also agreed that relations between India and Pakistan shall
be based on the principle of non-interference in the internal affairs of
each other.

They also agreed that both sides will discourage any propaganda
directed against the other country, and will encourage propaganda
which promotes the development of friendly relations between the two
countries.

Q Source: 12th TN History Textbook

73. The Mandal Commission Report 1980 was related to


a) Reservation for other backward classes
www.insightsonindia.com
www.insightsias.com
TEST 28 Solutions

b) Linguistic reorganization of Indian states


c) Design of Indias education policy
d) Development of Panchsheel principles

Solution: a)
Learning: The plan to set up another commission was taken by the Morarji
Desai government in 1978 as per the mandate of the under article 340 of the
constitution.

The commission estimated that 54% of the total population (excluding SCs
and STs), belonging to 3,743 different castes and communities, were
backward.

In 1980, the commission's report affirmed the affirmative action practice


under Indian law whereby members of lower castes (known as Other
Backward Classes (OBC), Scheduled Castes (SC) and Scheduled Tribes (ST))
were given exclusive access to a certain portion of government Jobs and slots
in public universities, and recommended changes to these quotas, increasing
them by 27% to 50%.

Q Source: Frequently in news due to OBC reservation issues

74. The Parliament can amend the Constitution subjected to the limit of
a) Altering the division of power between Centre and States
b) Basic Structure of the Constitution
c) Text of the Preamble
d) Directive Principles of State Policy

Solution: b)
Learning: In the Kesavananda Bharati case1 (1973), the Supreme Court ruled
that the constituent power of Parliament under Article 368 does not enable it
to alter the basic structure of the Constitution.

Options (a), (c) and (d) do not come in the basic structure. For e.g. Centre can
reduce the taxation powers of states under lets say GST, which is possible
through the Parliament.
www.insightsonindia.com
www.insightsias.com
TEST 28 Solutions

Preamble has already been amended once, and DPSP too has been extended.

Q Source: Indian Polity: M Laxmikanth

75. There are no tiger reserves in which state(s)?


1. Gujarat
2. Maharashtra
3. Andhra Pradesh
4. Himachal Pradesh

Select the correct answer using the codes below.

a) 1 and 3 only
b) 1 and 4 only
c) 2 only
d) 1, 2, 3 and 4

Solution: b)
Learning: The map below shows all important tiger reserves in India.

Go to the Q source to find out the list.

Gujarat and HP have other protected areas but not a tiger reserve.
www.insightsonindia.com
www.insightsias.com
TEST 28 Solutions

Q Source: http://wiienvis.nic.in/Database/trd_8222.aspx

76. The voting age for Indians was lowered to 18 under the Universal Adult
Franchise based on the
a) Parliamentary enactment immediately post-Independent
b) Unamended Indian Constitution adopted in 1950
c) Legislation backed by the Government in 1991
www.insightsonindia.com
www.insightsias.com
TEST 28 Solutions

d) None of the above

Solution: d)
Justification & Learning: The Indian Constitution adopts universal adult
franchise as a basis of elections to the Lok Sabha and the state legislative
assemblies.

Every citizen who is not less than 18 years of age has a right to vote without
any discrimination of caste, race, religion, sex, literacy, wealth, and so on. The
voting age was reduced to 18 years from 21 years in 1989 by the 61st
Constitutional Amendment Act of 1988.

The introduction of universal adult franchise by the Constitution-makers was


a bold experiment and highly remarkable in view of the vast size of the
country, its huge population, high poverty, social inequality and overwhelming
illiteracy.

Q Source: Indian Polity: M Laxmikanth

77. Multilingualism enables communication between the UN's linguistically


and culturally diverse Member States within UN, as UN has adopted a
number of official languages. Which of the following is NOT an official
language used at the United Nations (UN)?
a) Arabic
b) Hindi
c) Chinese
d) Spanish

Solution: b)
Learning: There are six official languages of the UN. These are Arabic,
Chinese, English, French, Russian and Spanish.

A delegate may speak in any official UN language. The speech is


interpreted simultaneously into the other official languages of the UN.

At times, a delegate may choose to make a statement using a non-official


language.
www.insightsonindia.com
www.insightsias.com
TEST 28 Solutions

In such cases, the delegation must provide either an interpretation or a


written text of the statement in one of the official languages.

Most UN documents are issued in all six official languages, requiring


translation from the original document.

Q Source: UN Website

78.As per Directive Principles in Part IV of the Constitution, the State shall
direct its policy towards securing that the ownership and control of the
material resources of the community so that
1. Concentration of wealth is avoided
2. Means of production are nationalized
3. Private ownership is gradually discouraged

Select the correct answer using the codes below.

a) 1 and 2 only
b) 2 only
c) 1 only
d) 2 and 3 only

Solution: c)
Justification: Article 39 (b) saysThe State shall direct its policy towards
securing that the ownership and control of the material resources of the
community are so distributed as best to subserve the common good.

Article 39 (c) saysThe state shall direct its policy towards securing that the
operation of the economic system does not result in the concentration of
wealth and means of production to the common detriment (it doesnt imply
either nationalization or shunning of private properties). So, both 2 and 3 will
be wrong.

Q Source: Indian Polity: M Laxmikanth


www.insightsonindia.com
www.insightsias.com
TEST 28 Solutions

79. Consider the following about Chemical Weapons Convention.


1. It bans the possession and development of Chemical weapons for
countries other than P-5 members of the UNSC.
2. India is a signatory to the convention.
3. It was negotiated at UN only after some concerns were raised about
the use of Chemical weapons in the Syrian Civil War.

Select the correct answer using the codes below.

a) 1 and 3 only
b) 2 only
c) 1 and 2 only
d) 1, 2 and 3

Solution: b)
Justification: Statement 1: It is a universal non-discriminatory, multilateral,
Disarmament Treaty which bans the development, production, acquisition,
transfer, use and stockpile of all chemical weapons. The Treaty puts all the
States Parties on an equal footing.

Countries having stockpiles of chemical weapons are required to declare and


destroy them.

Statement 2 and 3: The Convention was opened for signature on 13th January,
1993 in Paris. India signed the Convention on in January, 1993. Syrian Civil
war is recent post-Arab spring (2011 onwards).

Q Source: Ministry of chemicals & Fertilizers website

80. The Constitution deals with which of the following?


1. Adjudication of inter-state water disputes
2. Coordination between Centre and State through inter-state councils
3. Freedom of inter-state trade, commerce and intercourse
www.insightsonindia.com
www.insightsias.com
TEST 28 Solutions

Select the correct answer using the codes below.

a) 1 and 2 only
b) 3 only
c) 2 and 3 only
d) 1, 2 and 3

Solution: d)
Justification: Statement 1: Article 262 of the Constitution provides for the
adjudication of inter-state water disputes. It makes the provision that
Parliament may by law provide for the adjudication of any dispute or
complaint with respect to the use, distribution and control of waters of any
inter-state river and river valley.

Statement 2: Article 263 contemplates the establishment of an Inter-State


Council to effect coordination between the states and between Centre and
states.

Statement 3: Articles 301 to 307 in Part XIII of the Constitution deal with the
trade, commerce and intercourse within the territory of India.

Article 301 declares that trade, commerce and intercourse throughout the
territory of India shall be free.

Q Source: Indian Polity: M Laxmikanth

81. Which of the following point to the art form Mudiyettu?


a) It is a bamboo carving made by the use of natural colours giving it
a rich and sombre feel.
b) These are rock carvings inscribed on the temples of Chola
Kingdoms.
c) It is a form of rod puppetry practiced in the state of Andhra
Pradesh.
d) It is a ritual theatre and dance drama of Kerala.

Solution: d)
www.insightsonindia.com
www.insightsias.com
TEST 28 Solutions

Learning: It is one of the intangible heritages of India as accorded by


UNESCO.

It is performed for purification and rejuvenation of the community.


It is the enactment of the mythological tale of a battle between the
goddess Kali and the demon Darika.
The ritual is a part of the Bhagwati cult. It is performed in just 4 districts
of Kerela during the four day festival dedicated to Goddess Kali after the
summer harvest.
Musicians of Muddiyettu are maestros of Sopana Sangeeth, the mother
of Carnatic music in South India.
A complete Mudiyettu performance requires a total of 16 people.
The figure of Kali is also painted on the floor of the temple and then a
specific song is sung describing the goddess from head to toe.

Take a look at the short documentary made by UNESCO:

https://youtu.be/FEWXi2SHodk

Q Source: UNESCO India heritages

82. The Reserve Bank of India exercises its Developmental functions


by
1. Stepping up credit flow to MSMEs
2. Macro policy formulation to strengthen credit flow to the priority
sectors
3. Approving annual budget grants for weaker banks

Select the correct answer using the codes below.

a) 2 only
b) 1 and 2 only
c) 1 and 3 only
d) 1, 2 and 3

Solution: b)
www.insightsonindia.com
www.insightsias.com
TEST 28 Solutions

Learning: Statement 3 is wrong, as recapitalization of weak or stressed banks


is approved by the government. RBI is not an authority with respect to the GoI
budget.

RBI achieves development in the following way:

Financial inclusion - Credit flow to priority sectors: Macro policy


formulation to strengthen credit flow to the priority sectors. Ensuring
priority sector lending becomes a tool for banks for capturing untapped
business opportunities among the financially excluded sections of
society.

Financial inclusion and financial literacy: Help expand Prime Ministers


Jan Dhan Yojana (PMJDY) to become a sustainable and scalable
financial inclusion initiative.

Credit flow to MSME: Stepping up credit flow to micro, small and


medium enterprises (MSME) sector, rehabilitation of sick units through
timely credit support.

Institutions: Strengthening institutional arrangements, such as, State


Level Bankers Committees (SLBCs), Lead bank scheme, etc., to facilitate
achievement of above objectives.

Q Source: RBI Website

83. National Ambient Quality Standards as notified by the Central


Pollution Control Board (CPCB) does NOT cover which of the following
air pollutants?
a) Particulate Matter
b) Sulphur Dioxide
c) Ozone
d) Carbon Dioxide

Solution: d)
Learning: An order published in 2009 covers SO2, NO2, Lead, Ozone, PM
10, PM 2.5, CO, NH3 (Ammonia), Benzopyrene, Nickel and Arsenic.
www.insightsonindia.com
www.insightsias.com
TEST 28 Solutions

You can find the order here which was again put up recently on the website of
MoEF. http://cpcb.nic.in/National_Ambient_Air_Quality_Standards.php

These air pollutants are continuously monitored.

They were notified by Central Pollution Control Board

Q Source:
http://www.moef.nic.in/sites/default/files/notification/Recved%20national.p
df

84. Consider the following statements.


1. Warren Hastings established a Madrasa in Calcutta to encourage the
study of Muslim laws along with Arabic and Persian languages.
2. Jonathan Duncan established a Sanskrit College to promote the
study of Hindu laws and philosophy in Banaras.
3. William Bentinck announced a fixed annual sum for promotion of
vernacular languages and oriental studies.

Select the correct answer using the codes below.

a) 1 only
b) 2 and 3 only
c) 1 and 2 only
d) 1, 2 and 3

Solution: c)
Justification: The contemporary British scholars were divided into two
groups on the issue of development of education in India. One group, called
the Orientalists, advocated the promotion of oriental subjects through Indian
languages. The other group, called the Anglicists, argued the cause of western
sciences and literature in the medium of English language

Statement 1: Although the British had captured Bengal in 1757, yet the
responsibility of imparting education remained only in Indian hands.

The study of ancient texts written in Arabic, Persian and Sanskrit still
continued. In 1781, Warren Hastings established a Madrasa in Calcutta to
encourage the study of Muslim laws along with Arabic and Persian languages.
www.insightsonindia.com
www.insightsias.com
TEST 28 Solutions

Statement 2: A decade later in 1791 due to the sincere efforts of the British
resident, Jonathan Duncan, a Sanskrit College was established to promote the
study of Hindu laws and philosophy in Banaras.

Therefore, it must be contended that during the first three decades of the 19th
century, the development of education took place only through the traditional
institutions.

Statement 3: He discouraged oriental studies. In 1829, after assuming the


office of the Governor-General of India, Lord William Bentinck, emphasized
on the medium of English language in Indian education.

Q Source: 12th TN History Textbook

85. Firewall is a term associated with


a) Computing
b) Agriculture
c) Aerodynamics
d) Mining

Solution: a)
Learning: A firewall is a network security system, either hardware- or
software-based, that controls incoming and outgoing network traffic based on
a set of rules.

A firewall typically establishes a barrier between a trusted, secure internal


network and another outside network, such as the Internet, that is assumed
not to be secure or trusted.

The image below is a visualization.


www.insightsonindia.com
www.insightsias.com
TEST 28 Solutions

Q Source: DeITY Website

86. Highest weightage in the Index of Industrial Production (IIP) is


carried by
a) Mining
b) Manufacturing
c) Electricity
d) Fertilizers

Solution: b)
Learning: The weightage of Manufacturing, Mining and Electricity
production in overall Index of Industrial Production (IIP) is 75.52 %, 14.16 %
and 10.32 % respectively.

The Index of Industrial Production (IIP) is an index which shows the growth
rates in different industry groups of the economy in a stipulated period of
time. The IIP index is computed and published by the Central Statistical
Organisation (CSO) on a monthly basis.

Q Source: Economic Adviser: DIPP Website


www.insightsonindia.com
www.insightsias.com
TEST 28 Solutions

87.Wildlife Trust of India (WTI) is a/an


1. Statutory body under the Ministry of Environment, Forests and
Climate Change
2. Organization that works towards securing the natural heritage of
India

Which of the above is/are correct?

a) 1 only
b) 2 only
c) Both 1 and 2
d) None

Solution: b)
Justification: Statement 1 and 2: Wildlife Trust of India (WTI) is a leading
Indian nature conservation organisation committed to the service of nature.

Its mission is to conserve wildlife and its habitat and to work for the welfare of
individual wild animals, in partnership with communities and governments.

WTIs team of 150 dedicated professionals work towards achieving its vision of
a secure natural heritage of India, in six priority landscapes, knit holistically
together by seven key strategies or Big Ideas.

Q Source: MoEF Website

88. Plant quarantine is vital to prevent the introduction of


1. Non-indigenous potentially damaging pests in the country
2. Diseases of plants in host country

Which of the above is/are correct?

a) 1 only
b) 2 only
c) Both 1 and 2
d) None

Solution: c)
www.insightsonindia.com
www.insightsias.com
TEST 28 Solutions

Justification: Less-developed countries and other countries in transition


are especially vulnerable to the damaging effects of exotic pest introductions
because of often inadequate infrastructure and the fragility of their economies.

Plant quarantine is a technique for ensuring disease- and pest-free plants,


whereby a plant is isolated while tests are performed to detect the presence of
a problem.

It is crucial to prevent the introduction of non-indigenous, potentially


damaging pests and diseases of plants into a country or to eradicate them
before they can become widespread and well established.

Q Source: Agritech TNAU Website

89. Muga Silk culture is specific to the state of X and an integral


part of the tradition and culture of that state. The State X is
a) Uttar Pradesh
b) Assam
c) Nagaland
d) Jharkhand

Solution: b)
Learning: Muga is a golden yellow colour silk and the pride of Assam state.

It is obtained from semi-domesticated multivoltine silkworm, Antheraea


assamensis.

These silkworms feed on the aromatic leaves of Som and Soalu plants and are
reared on trees similar to that of tasar.

Muga culture is specific to the state of Assam.

The muga silk, a high value product is used in products like sarees, mekhalas,
chaddars, etc.

Q Source: Central Silk Board Website


www.insightsonindia.com
www.insightsias.com
TEST 28 Solutions

90. World Energy Outlook Report is published by


a) World Bank
b) United Nations Framework Convention on Climate Change
(UNFCCC)
c) International Energy Agency (IEA)
d) United Nations Environment Program (UNEP)

Solution: c)
Learning: The IEA is an autonomous organisation which works to ensure
reliable, affordable and clean energy for its 29 member countries and beyond.
The IEA has four main areas of focus: energy security, economic development,
environmental awareness and engagement worldwide.

As per IEA website, The annual World Energy Outlook (WEO) is now the
worlds most authoritative source of energy market analysis and projections,
providing critical analytical insights into trends in energy demand and supply
and what they mean for energy security, environmental protection and
economic development.

The WEO projections are used by the public and private sector as a framework
on which they can base their policy-making, planning and investment
decisions and to identify what needs to be done to arrive at a supportable and
sustainable energy future.

Q Source: IEA Website

91. Who among the following is generally the leader of the Lok Sabha?
a) Speaker, Lok Sabha
b) Leader of Opposition
c) Prime Minister
d) Secretary General, Lok Sabha

Solution: c)
www.insightsonindia.com
www.insightsias.com
TEST 28 Solutions

Learning: As per LS rules of procedure and definitions, Leader of the


House means the Prime Minister, if Prime Minister is a member of the
House, or a Minister who is a member of the House and is nominated by the
Prime Minister to function as the Leader of the House.

In whichever case, the leader will belong to the largest party/coalition of the
house.

Q Source: http://164.100.47.192/loksabha/rules.aspx

92. The boundary of the national park is from the point where the
Sankosh river cuts the Indo-Bhutan international boundary. The
national park is
a) Neora national park
b) Namdapha national park
c) Digha national park
d) Manas national park

Solution: d)
Learning: Manas extends over an area of 2837 Sq. Km from Sankosh river in
the west to Dhansiri river in the east, with a core area of 500 Sq. Km. of the
National park, which declared in 1990.

The average elevation of the area is 85 m above mean sea level. The river
Manas flows into the national Park from the gorges of Bhutan and split into
two major streams of which the main water course comes out of the National
Park about 30 km downstream is known as Beki.

About the half of the Park is covered by Grasslands of Terai and Bhabar type,
the riparian areas have colonizing grasslands and woodlands of several
species. The thick woodlands are called Eastern Moist Deciduous Forests of
various types. The undergrowths are very thick.

Q Source: MoEF Website


www.insightsonindia.com
www.insightsias.com
TEST 28 Solutions

93. National Pharmaceutical Pricing Authority (NPPA) can


1. Fix the prices of controlled bulk drugs and formulations
2. Enforce prices and availability of the medicines in the country

Which of the above is/are correct?

a) 1 only
b) 2 only
c) Both 1 and 2
d) None

Solution: c)
Justification: Statement 1: NPPA is an organization of the Government of
India which was established, inter alia, to fix/ revise the prices of controlled
bulk drugs and formulations and to enforce prices and availability of the
medicines in the country, under the Drugs (Prices Control) Order, 1995.

Statement 2: The organization is also entrusted with the task of recovering


amounts overcharged by manufacturers for the controlled drugs from the
consumers.

It also monitors the prices of decontrolled drugs in order to keep them at


reasonable levels.

Q Source: http://pharmaceuticals.gov.in/important-link

94. Consider the following about Anticyclones.


1. These are low pressure regions.
2. Their extent is always small.
3. Cloudy and precipitation conditions exist alongwith the cyclone.

Select the correct answer using the codes below.

a) 1 and 2 only
b) 2 only
c) 3 only
d) 1, 2 and 3

Solution: c)
www.insightsonindia.com
www.insightsias.com
TEST 28 Solutions

Justification: Statement 1: An anti-cyclone -- also known as a high pressure


area -- is a large atmospheric circulation system with the wind flowing
clockwise around it in the Northern Hemisphere, and counter-clockwise in the
Southern Hemisphere.

Statement 2: The extents are generally large and the effects are also
pronounced.

Statement 3: Anticyclones form from air masses cooling more than their
surroundings, which causes the air to contract slightly making the air more
dense. Since dense air weighs more, the weight of the atmosphere overlying a
location increases, causing increased surface air pressure.

Winds are of moderate speed, and at the outer edges, cloudy and precipitation
conditions exist.

Q Source: Improvisation: 11th NCERT: Fundamentals of Physical Geography

95.Which of the following is/are the functions of the Central Social Welfare
Board (CSWB)?
1. Implementing welfare programmes for women and children through
voluntary organizations
2. Approving all major welfare schemes of Ministry of Social Justice
and Empowerment

Which of the above is/are correct?

a) 1 only
b) 2 only
c) Both 1 and 2
d) None

Solution: a)
Justification: The Central Social Welfare Board (CSWB) was set-up in 1953
with the objective of promoting social welfare activities and implementing
welfare programmes for women and children through voluntary organizations.

Thirty-three state social welfare boards were set up with jurisdiction


over all states and union territories to work together with Central Social
www.insightsonindia.com
www.insightsias.com
TEST 28 Solutions

Welfare Board in implementing the programmes of the Board across the


country.

It is functioning as an interface between the government and the


voluntary sector.

The Board has been making concerted efforts for empowerment of


women through family counselling programmes, short stay homes,
awareness generation projects, condensed Course of Education
Programmes and other support services. The Board was registered as a
Charitable Company in 1969.

Q Source: India Yearbook 2016

96. Micro-propagation is widely used in forestry and in floriculture. It


can be used for
1. Conserving rare or endangered plant species
2. Obtaining a large number of plants within a short time

Which of the above is/are correct?

a) 1 only
b) 2 only
c) Both 1 and 2
d) None

Solution: c)
Justification: Plants can be propagated by sexual (through generation of
seeds) or asexual (through multiplication of vegetative parts) means.

Clonal (micro) propagation refers to the process of asexual reproduction by


multiplication of genetically identical copies of individual plants.

Micro propagation has become a suitable alternative to conventional methods


of vegetative propagation of plants. There are several advantages of micro
propagation.

Through micro propagation, a large number of plants can be grown


from a piece of plant tissue within a short period.
www.insightsonindia.com
www.insightsias.com
TEST 28 Solutions

Another advantage is that micro propagation can be carried out


throughout the year, irrespective of the seasonal variations.

It is also possible to produce disease-free plants through micro


propagation.

Micro propagation requires minimum growing space. Thus, millions of


plant species can be maintained inside culture vials in a small room in a
nursery.

Q Source: Agritech TNAU Website

97. Consider the following statements about Eastern Ghats.


1. They do not extend to the state of Odisha and Karnataka being
limited to Tamil Nadu and Andhra Pradesh.
2. They are eroded and cut through by Chambal and Mahanadi among
other Peninsular rivers.

Which of the above is/are correct?

a) 1 only
b) 2 only
c) Both 1 and 2
d) None

Solution: d)
Justification: Statement 1: The Eastern Ghats run from West Bengal state
in the north, through Odisha and Andhra Pradesh to Tamil Nadu in the south
passing some parts of Karnataka.

Statement 2: Chambal forms part of the Gangetic drainage system, so it cant


cut Eastern Ghats which are not spread till the Ganges drainage system.

They are eroded and cut through by the four major rivers of peninsular India,
known as the Godavari, Mahanadi, Krishna, and Kaveri.

Learning: The Eastern Ghats are a discontinuous range of mountains along


India's eastern coast.
www.insightsonindia.com
www.insightsias.com
TEST 28 Solutions

The mountain ranges run parallel to the Bay of Bengal.

The Deccan Plateau lies to the west of the range, between the Eastern Ghats
and Western Ghats.

The coastal plains, including the Coromandel Coast region, lie between the
Eastern Ghats and the Bay of Bengal. The Eastern Ghats are not as high as the
Western Ghats.

Q Source: NCERT 9th Geography

98. The Ilbert Bill Agitation 1884 was related to


a) Reservations of minorities in British India
b) Heavy military spending to capture foreign territories
c) Discrimination between Indians and Europeans in the
administration of criminal justice
d) Powers given to British to seize arms without notice

Solution: c)
Learning: Lord Ripon wanted to remove two kinds of law that had been
prevalent in India. According to the system of law, a European could be tried
only by a European Judge or a European Magistrate.

The disqualification was unjust and it was sought to cast a needless discredit
and dishonour upon the Indian-born members of the judiciary.

C.P. Ilbert, Law Member, introduced a bill in 1883 to abolish this


discrimination in judiciary. But Europeans opposed this Bill strongly.

Q Source: 12th TN History Textbook

99. The price of petroleum products has been deregulated some time
back with the primary aim to
a) Encourage the use of public transport
www.insightsonindia.com
www.insightsias.com
TEST 28 Solutions

b) Reduce oil imports in the country


c) Reduce the burden of subsidies given to the oil companies
d) Curb the use of black money in the economy

Solution: c)
Learning: The amount of subsidy outgo on these products was huge.

In the first quarter of this fiscal (April-June 2014), the under-recovery burden
on oil marketing companies was Rs 9,037 crore which would have required a
sharing mechanism between the budget, consumers, OMCs and upstream oil
and gas companies.

The increase in prices would reduce subsidy burden, make production from
stranded and deeper gas pockets more viable and boost domestic supply,
thereby attracting investment but without hurting its consumers like power
and fertiliser and the common man which uses piped gas for cooking or for
filling up their automobiles.

Q Source: http://www.thehindu.com/business/Economy/diesel-prices-
deregulated/article6514970.ece

100. The Government is promoting National Smart Grids Mission


(NSGM) to give a fillip to the power sector for its modernization. Which
of the following best describes a smart grid?
a) Any power grid that runs only on renewable energy
b) Power grids with open access and free user charges with access to
remote areas
c) Grids that allow for two-way communication between the power
utility and its customers for better power management
d) Grids that are wireless and do not require any power source

Solution: c)
Learning: The digital technology that allows for two-way communication
between the utility and its customers, and the sensing along the transmission
lines is what makes the grid smart.
www.insightsonindia.com
www.insightsias.com
TEST 28 Solutions

Like the Internet, the Smart Grid will consist of controls, computers,
automation, and new technologies and equipment working together, but in
this case, these technologies will work with the electrical grid to respond
digitally to our quickly changing electric demand.

The benefits associated with the Smart Grid include:

More efficient transmission of electricity; quicker restoration of


electricity after power disturbances; reduced operations and
management costs for utilities, and ultimately lower power costs for
consumers

Reduced peak demand, which will also help lower electricity rates;
Increased integration of large-scale renewable energy systems

; Better integration of customer-owner power generation systems, including


renewable energy systems and improved security

Because of its two-way interactive capacity, the Smart Grid will allow for
automatic rerouting when equipment fails or outages occur (see here how
https://www.smartgrid.gov/the_smart_grid/smart_grid.html - explanation
take from here)

This will minimize outages and minimize the effects when they do
happen. When a power outage occurs, Smart Grid technologies will
detect and isolate the outages, containing them before they become
large-scale blackouts.

The new technologies will also help ensure that electricity recovery
resumes quickly and strategically after an emergency.

Q Source:
http://powermin.nic.in/sites/default/files/uploads/National_Smart_Grid_M
ission_OM.pdf


www.insightsonindia.com
www.insightsias.com
TEST 28 Solutions

Vous aimerez peut-être aussi